Prove that A = x : φx = λx.42 is not recursive; i.e., x ∈ A is unsolvable/undecidable. [closed]Show that the language TOT= M is a Turing Machine that halts with all inputs is not recursively enumerable nor its complement.Prove that $overlineL$ is not recognizable by showing that $B_TM le_m L$Clarification of the argument for the set of total recursive functions not being recursively enumerable?Do there exist infinite sets of non-halting programs such that every program in the set computes every other program in the set?How to argue that a set is recursive or recursively enumerable?Using reductions of turing machines properlyWhy do we get a contradiction for $x=n$?Non recursive set that intersects every recursively enumerable setProve that set of all perfect numbers is primitive recursiveFunctions corresponding to Turing machines that might not halt but consume bounded tape

Mixing PEX brands

Redundant comparison & "if" before assignment

Strong empirical falsification of quantum mechanics based on vacuum energy density

Why does AES have exactly 10 rounds for a 128-bit key, 12 for 192 bits and 14 for a 256-bit key size?

Unexpected behavior of the procedure `Area` on the object 'Polygon'

Why did the EU agree to delay the Brexit deadline?

What is Cash Advance APR?

Can a Canadian Travel to the USA twice, less than 180 days each time?

Does the Linux kernel need a file system to run?

Does an advisor owe his/her student anything? Will an advisor keep a PhD student only out of pity?

Why is the "ls" command showing permissions of files in a FAT32 partition?

How to hide some fields of struct in C?

Angel of Condemnation - Exile creature with second ability

Store Credit Card Information in Password Manager?

How does a computer interpret real numbers?

Is aluminum electrical wire used on aircraft?

Can a stoichiometric mixture of oxygen and methane exist as a liquid at standard pressure and some (low) temperature?

Does IPv6 have similar concept of network mask?

Multiplicative persistence

What exact color does ozone gas have?

Calculating total slots

Did arcade monitors have same pixel aspect ratio as TV sets?

How do apertures which seem too large to physically fit work?

Is there a RAID 0 Equivalent for RAM?



Prove that A = x : φx = λx.42 is not recursive; i.e., x ∈ A is unsolvable/undecidable. [closed]


Show that the language TOT= M is a Turing Machine that halts with all inputs is not recursively enumerable nor its complement.Prove that $overlineL$ is not recognizable by showing that $B_TM le_m L$Clarification of the argument for the set of total recursive functions not being recursively enumerable?Do there exist infinite sets of non-halting programs such that every program in the set computes every other program in the set?How to argue that a set is recursive or recursively enumerable?Using reductions of turing machines properlyWhy do we get a contradiction for $x=n$?Non recursive set that intersects every recursively enumerable setProve that set of all perfect numbers is primitive recursiveFunctions corresponding to Turing machines that might not halt but consume bounded tape













-1












$begingroup$


please help to understand. Here is a similar example:
hint: (via S-m-n) that
K ≤ A.
Definition. (Complete Index Sets) Let C ⊆ P and A = x : φx ∈ C.
A is thus the set of ALL programs (known by their addresses) x that compute
any unary f ∈ C: Indeed, let λx.f(x) ∈ C. Thus f = φi for some i. Then i ∈ A.
But this is true of all φm that equal f.
We call A a complete (all) index (programs) set.
example:
The set A = x : ran(φx) = ∅ is not semi-recursive.
Recall that “range” for λx.f(x), denoted by ran(f), is defined by
x : (∃y)f(y) = x
We will try to show that
K ≤ A
ψ(x, y) = (
0 if φx(x) ↓
↑ if φx(x) ↑
Here is how to compute ψ:
Given x, y, ignore y. Fetch machine M at address x from the standard listing,
and call it on input x. If it ever halts, then print “0” and halt everything. If
it never halts, then you will never return from the call, which is the correct
specified in (2) behavior for ψ(x, y).
By CT, ψ is in P, so, by the S-m-n Theorem, there is a recursive h such that
ψ(x, y) = φh(x)(y), for all x, y
You may NOT use S-m-n UNTIL after you have proved that your
“λxy.ψ(x, y)” is in P (partial recursive).
We can rewrite this as,
φh(x)(y) = (
0 if φx(x) ↓
↑ if φx(x) ↑
or, rewriting (3) without arguments (as equality of functions, not equality of
function calls)
φh(x) =



{λy.0 if φx(x) ↓
{∅ if φx(x) ↑



In 3, ∅ stands for λy. ↑, the empty function.
Thus,
h(x) ∈ A iff ran(φh(x)) = ∅
bottom case in
h(x) ∈ A iff ran(φh(x)) = ∅ iff φx(x) ↑
The above says x ∈ K ≡ h(x) ∈ A, hence K ≤ A and
thus A (don't belong to) P.










share|cite|improve this question









$endgroup$



closed as unclear what you're asking by Shailesh, mrtaurho, Vinyl_cape_jawa, José Carlos Santos, Riccardo.Alestra Mar 15 at 15:03


Please clarify your specific problem or add additional details to highlight exactly what you need. As it's currently written, it’s hard to tell exactly what you're asking. See the How to Ask page for help clarifying this question. If this question can be reworded to fit the rules in the help center, please edit the question.













  • 1




    $begingroup$
    A reference for MathJax, a means of formatting your math text and such on this site -- math.meta.stackexchange.com/questions/5020/…
    $endgroup$
    – Eevee Trainer
    Mar 15 at 3:04















-1












$begingroup$


please help to understand. Here is a similar example:
hint: (via S-m-n) that
K ≤ A.
Definition. (Complete Index Sets) Let C ⊆ P and A = x : φx ∈ C.
A is thus the set of ALL programs (known by their addresses) x that compute
any unary f ∈ C: Indeed, let λx.f(x) ∈ C. Thus f = φi for some i. Then i ∈ A.
But this is true of all φm that equal f.
We call A a complete (all) index (programs) set.
example:
The set A = x : ran(φx) = ∅ is not semi-recursive.
Recall that “range” for λx.f(x), denoted by ran(f), is defined by
x : (∃y)f(y) = x
We will try to show that
K ≤ A
ψ(x, y) = (
0 if φx(x) ↓
↑ if φx(x) ↑
Here is how to compute ψ:
Given x, y, ignore y. Fetch machine M at address x from the standard listing,
and call it on input x. If it ever halts, then print “0” and halt everything. If
it never halts, then you will never return from the call, which is the correct
specified in (2) behavior for ψ(x, y).
By CT, ψ is in P, so, by the S-m-n Theorem, there is a recursive h such that
ψ(x, y) = φh(x)(y), for all x, y
You may NOT use S-m-n UNTIL after you have proved that your
“λxy.ψ(x, y)” is in P (partial recursive).
We can rewrite this as,
φh(x)(y) = (
0 if φx(x) ↓
↑ if φx(x) ↑
or, rewriting (3) without arguments (as equality of functions, not equality of
function calls)
φh(x) =



{λy.0 if φx(x) ↓
{∅ if φx(x) ↑



In 3, ∅ stands for λy. ↑, the empty function.
Thus,
h(x) ∈ A iff ran(φh(x)) = ∅
bottom case in
h(x) ∈ A iff ran(φh(x)) = ∅ iff φx(x) ↑
The above says x ∈ K ≡ h(x) ∈ A, hence K ≤ A and
thus A (don't belong to) P.










share|cite|improve this question









$endgroup$



closed as unclear what you're asking by Shailesh, mrtaurho, Vinyl_cape_jawa, José Carlos Santos, Riccardo.Alestra Mar 15 at 15:03


Please clarify your specific problem or add additional details to highlight exactly what you need. As it's currently written, it’s hard to tell exactly what you're asking. See the How to Ask page for help clarifying this question. If this question can be reworded to fit the rules in the help center, please edit the question.













  • 1




    $begingroup$
    A reference for MathJax, a means of formatting your math text and such on this site -- math.meta.stackexchange.com/questions/5020/…
    $endgroup$
    – Eevee Trainer
    Mar 15 at 3:04













-1












-1








-1





$begingroup$


please help to understand. Here is a similar example:
hint: (via S-m-n) that
K ≤ A.
Definition. (Complete Index Sets) Let C ⊆ P and A = x : φx ∈ C.
A is thus the set of ALL programs (known by their addresses) x that compute
any unary f ∈ C: Indeed, let λx.f(x) ∈ C. Thus f = φi for some i. Then i ∈ A.
But this is true of all φm that equal f.
We call A a complete (all) index (programs) set.
example:
The set A = x : ran(φx) = ∅ is not semi-recursive.
Recall that “range” for λx.f(x), denoted by ran(f), is defined by
x : (∃y)f(y) = x
We will try to show that
K ≤ A
ψ(x, y) = (
0 if φx(x) ↓
↑ if φx(x) ↑
Here is how to compute ψ:
Given x, y, ignore y. Fetch machine M at address x from the standard listing,
and call it on input x. If it ever halts, then print “0” and halt everything. If
it never halts, then you will never return from the call, which is the correct
specified in (2) behavior for ψ(x, y).
By CT, ψ is in P, so, by the S-m-n Theorem, there is a recursive h such that
ψ(x, y) = φh(x)(y), for all x, y
You may NOT use S-m-n UNTIL after you have proved that your
“λxy.ψ(x, y)” is in P (partial recursive).
We can rewrite this as,
φh(x)(y) = (
0 if φx(x) ↓
↑ if φx(x) ↑
or, rewriting (3) without arguments (as equality of functions, not equality of
function calls)
φh(x) =



{λy.0 if φx(x) ↓
{∅ if φx(x) ↑



In 3, ∅ stands for λy. ↑, the empty function.
Thus,
h(x) ∈ A iff ran(φh(x)) = ∅
bottom case in
h(x) ∈ A iff ran(φh(x)) = ∅ iff φx(x) ↑
The above says x ∈ K ≡ h(x) ∈ A, hence K ≤ A and
thus A (don't belong to) P.










share|cite|improve this question









$endgroup$




please help to understand. Here is a similar example:
hint: (via S-m-n) that
K ≤ A.
Definition. (Complete Index Sets) Let C ⊆ P and A = x : φx ∈ C.
A is thus the set of ALL programs (known by their addresses) x that compute
any unary f ∈ C: Indeed, let λx.f(x) ∈ C. Thus f = φi for some i. Then i ∈ A.
But this is true of all φm that equal f.
We call A a complete (all) index (programs) set.
example:
The set A = x : ran(φx) = ∅ is not semi-recursive.
Recall that “range” for λx.f(x), denoted by ran(f), is defined by
x : (∃y)f(y) = x
We will try to show that
K ≤ A
ψ(x, y) = (
0 if φx(x) ↓
↑ if φx(x) ↑
Here is how to compute ψ:
Given x, y, ignore y. Fetch machine M at address x from the standard listing,
and call it on input x. If it ever halts, then print “0” and halt everything. If
it never halts, then you will never return from the call, which is the correct
specified in (2) behavior for ψ(x, y).
By CT, ψ is in P, so, by the S-m-n Theorem, there is a recursive h such that
ψ(x, y) = φh(x)(y), for all x, y
You may NOT use S-m-n UNTIL after you have proved that your
“λxy.ψ(x, y)” is in P (partial recursive).
We can rewrite this as,
φh(x)(y) = (
0 if φx(x) ↓
↑ if φx(x) ↑
or, rewriting (3) without arguments (as equality of functions, not equality of
function calls)
φh(x) =



{λy.0 if φx(x) ↓
{∅ if φx(x) ↑



In 3, ∅ stands for λy. ↑, the empty function.
Thus,
h(x) ∈ A iff ran(φh(x)) = ∅
bottom case in
h(x) ∈ A iff ran(φh(x)) = ∅ iff φx(x) ↑
The above says x ∈ K ≡ h(x) ∈ A, hence K ≤ A and
thus A (don't belong to) P.







computability recursive-algorithms turing-machines






share|cite|improve this question













share|cite|improve this question











share|cite|improve this question




share|cite|improve this question










asked Mar 15 at 3:03









Alexey UgadovAlexey Ugadov

1




1




closed as unclear what you're asking by Shailesh, mrtaurho, Vinyl_cape_jawa, José Carlos Santos, Riccardo.Alestra Mar 15 at 15:03


Please clarify your specific problem or add additional details to highlight exactly what you need. As it's currently written, it’s hard to tell exactly what you're asking. See the How to Ask page for help clarifying this question. If this question can be reworded to fit the rules in the help center, please edit the question.









closed as unclear what you're asking by Shailesh, mrtaurho, Vinyl_cape_jawa, José Carlos Santos, Riccardo.Alestra Mar 15 at 15:03


Please clarify your specific problem or add additional details to highlight exactly what you need. As it's currently written, it’s hard to tell exactly what you're asking. See the How to Ask page for help clarifying this question. If this question can be reworded to fit the rules in the help center, please edit the question.









  • 1




    $begingroup$
    A reference for MathJax, a means of formatting your math text and such on this site -- math.meta.stackexchange.com/questions/5020/…
    $endgroup$
    – Eevee Trainer
    Mar 15 at 3:04












  • 1




    $begingroup$
    A reference for MathJax, a means of formatting your math text and such on this site -- math.meta.stackexchange.com/questions/5020/…
    $endgroup$
    – Eevee Trainer
    Mar 15 at 3:04







1




1




$begingroup$
A reference for MathJax, a means of formatting your math text and such on this site -- math.meta.stackexchange.com/questions/5020/…
$endgroup$
– Eevee Trainer
Mar 15 at 3:04




$begingroup$
A reference for MathJax, a means of formatting your math text and such on this site -- math.meta.stackexchange.com/questions/5020/…
$endgroup$
– Eevee Trainer
Mar 15 at 3:04










0






active

oldest

votes

















0






active

oldest

votes








0






active

oldest

votes









active

oldest

votes






active

oldest

votes

Popular posts from this blog

Lowndes Grove History Architecture References Navigation menu32°48′6″N 79°57′58″W / 32.80167°N 79.96611°W / 32.80167; -79.9661132°48′6″N 79°57′58″W / 32.80167°N 79.96611°W / 32.80167; -79.9661178002500"National Register Information System"Historic houses of South Carolina"Lowndes Grove""+32° 48' 6.00", −79° 57' 58.00""Lowndes Grove, Charleston County (260 St. Margaret St., Charleston)""Lowndes Grove"The Charleston ExpositionIt Happened in South Carolina"Lowndes Grove (House), Saint Margaret Street & Sixth Avenue, Charleston, Charleston County, SC(Photographs)"Plantations of the Carolina Low Countrye

random experiment with two different functions on unit interval Announcing the arrival of Valued Associate #679: Cesar Manara Planned maintenance scheduled April 23, 2019 at 00:00UTC (8:00pm US/Eastern)Random variable and probability space notionsRandom Walk with EdgesFinding functions where the increase over a random interval is Poisson distributedNumber of days until dayCan an observed event in fact be of zero probability?Unit random processmodels of coins and uniform distributionHow to get the number of successes given $n$ trials , probability $P$ and a random variable $X$Absorbing Markov chain in a computer. Is “almost every” turned into always convergence in computer executions?Stopped random walk is not uniformly integrable

How should I support this large drywall patch? Planned maintenance scheduled April 23, 2019 at 00:00UTC (8:00pm US/Eastern) Announcing the arrival of Valued Associate #679: Cesar Manara Unicorn Meta Zoo #1: Why another podcast?How do I cover large gaps in drywall?How do I keep drywall around a patch from crumbling?Can I glue a second layer of drywall?How to patch long strip on drywall?Large drywall patch: how to avoid bulging seams?Drywall Mesh Patch vs. Bulge? To remove or not to remove?How to fix this drywall job?Prep drywall before backsplashWhat's the best way to fix this horrible drywall patch job?Drywall patching using 3M Patch Plus Primer